Difference between revisions of "2021 Fall AMC 10B Problems/Problem 22"

(Solution 1)
(Problem)
Line 1: Line 1:
 
==Problem==
 
==Problem==
 
For each integer <math> n\geq 2 </math>, let <math> S_n </math> be the sum of all products <math> jk </math>, where <math> j </math> and <math> k </math> are integers and <math> 1\leq j<k\leq n </math>. What is the sum of the 10 least values of <math> n </math> such that <math> S_n </math> is divisible by <math> 3 </math>?
 
For each integer <math> n\geq 2 </math>, let <math> S_n </math> be the sum of all products <math> jk </math>, where <math> j </math> and <math> k </math> are integers and <math> 1\leq j<k\leq n </math>. What is the sum of the 10 least values of <math> n </math> such that <math> S_n </math> is divisible by <math> 3 </math>?
 +
 
<math>\textbf{(A)}\ 196\qquad\textbf{(B)}\ 197\qquad\textbf{(C)}\ 198\qquad\textbf{(D)}\ 199\qquad\textbf{(E)}\ 200</math>
 
<math>\textbf{(A)}\ 196\qquad\textbf{(B)}\ 197\qquad\textbf{(C)}\ 198\qquad\textbf{(D)}\ 199\qquad\textbf{(E)}\ 200</math>
  

Revision as of 11:22, 23 November 2021

Problem

For each integer $n\geq 2$, let $S_n$ be the sum of all products $jk$, where $j$ and $k$ are integers and $1\leq j<k\leq n$. What is the sum of the 10 least values of $n$ such that $S_n$ is divisible by $3$?

$\textbf{(A)}\ 196\qquad\textbf{(B)}\ 197\qquad\textbf{(C)}\ 198\qquad\textbf{(D)}\ 199\qquad\textbf{(E)}\ 200$

Solution 1

To get from $S_n$ to $S_{n+1}$, we add $1(n+1)+2(n+1)+\cdots +n(n+1)=(1+2+\cdots +n)(n+1)=\frac{n(n+1)^2}{2}$.

Now, we can look at the different values of $n$ mod $3$. For $n\equiv 0\pmod{3}$ and $n\equiv 2\pmod{3}$, then we have $\frac{n(n+1)^2}{2}\equiv 0\pmod{3}$. However, for $n\equiv 1\pmod{3}$, we have \[\frac{1\cdot {2}^2}{2}\equiv 2\pmod{3}.\]

Clearly, $S_2\equiv 2\pmod{3}.$ Using the above result, we have $S_5\equiv 1\pmod{3}$, and $S_8$, $S_9$, and $S_{10}$ are all divisible by $3$. After $3\cdot 3=9$, we have $S_{17}$, $S_{18}$, and $S_{19}$ all divisible by $3$, as well as $S_{26}, S_{27}, S_{28}$, and $S_{35}$. Thus, our answer is $8+9+10+17+18+19+26+27+28+35=27+54+81+35=162+35=\boxed{\mathrm{(B)}\ 197}$. -BorealBear

Solution 2 (bash)

See Also

2021 Fall AMC 10B (ProblemsAnswer KeyResources)
Preceded by
Problem 21
Followed by
Problem 23
1 2 3 4 5 6 7 8 9 10 11 12 13 14 15 16 17 18 19 20 21 22 23 24 25
All AMC 10 Problems and Solutions

The problems on this page are copyrighted by the Mathematical Association of America's American Mathematics Competitions. AMC logo.png